Emergency Medicine Genitourinary

¡Supera tus tareas y exámenes ahora con Quizwiz!

One Step Further Question: What are the classic indications for emergent dialysis?

Answer: "AEIOU": A-acidosis; E-electrolyte abnormalities (e.g., hyperkalemia); I-ingestion of toxic dialyzable substances (e.g., lithium, methanol); O-overload of fluid; U-uremia.

One Step Further Question: What is the probability that a 3-mm stone will pass spontaneously?

Answer: 90%.

One Step Further Question: What BUN and creatinine are considered triggers for the initiation of dialysis in patients with CKD?

Answer: A BUN of 100 mg/dl and a Cr of 10 mg/dl.

One Step Further Question: Aside from BUN/Creatinine ratio, what other lab findings are consistent with prerenal acute renal failure?

Answer: A Urine Sodium <20 mEq/L and a Fraction of Excreted Na (FENa) <1%

One Step Further Question: How common is amiodarone induced epididymitis?

Answer: About 3-11% of patients receiving amiodarone can develop epididymitis.

One Step Further Question: What is the another name for acute tubular necrosis (ATN)?

Answer: Acute Tubular Injury (ATI).

One Step Further Question: Why is phimosis less common as patients age?

Answer: At birth, only 4% of boys have a fully retractable foreskin. Physiologic changes increase this rate until 4 years of age when 90% are fully retractable.

One Step Further Question: What age is testicular torsion most likely to occur?

Answer: Bimodal distribution with peak incidence in the neonate within first few days of life and in preadolescence

One Step Further Question: What is the most common type of kidney stone?

Answer: Calcium Oxalate.

One Step Further Question: What are the two most common organisms responsible for epididymitis under the age of 35?

Answer: Chlamydia trachomatis and Neisseria gonorrhea.

One Step Further Question: What is the most common bacterial cause of UTI?

Answer: E. coli.

One Step Further Question: What is first-line therapy in a child with idiopathic nephrotic syndrome?

Answer: Glucocorticoids.

One Step Further Question: Do antibiotics reduce the incidence of poststreptococcal glomerulonephritis?

Answer: No.

One Step Further Question: Name the 5 anatomic locations in which renal stones become impacted.

Answer: Renal calyx, ureteropelvic junction, pelvic brim, ureterovesicular junction, and vesicle orifice.

One Step Further Question: What is the most common cause of ATN?

Answer: Renal ischemia from sepsis, surgery, or trauma.

One Step Further Question: What other bacterial agents are commonly associated with reactive arthritis?

Answer: Shigella, Salmonella, Yersinia and Campylobacter organisms.

One Step Further Question: What emergent procedure needs to be performed in a patient with paraphimosis when all reduction techniques have failed and ongoing ischemia is clearly evident?

Answer: Superficial dorsal slit incision of the constricting band.

One Step Further Question: What is the third most common cause of genital ulcers in addition to chancroid and herpes?

Answer: Syphilis.

One Step Further Question: What test is used to diagnose a hydrocele?

Answer: Transillumination.

One Step Further Question: What is the utility of a Tzanck test?

Answer: Tzanck tests are no longer recommended due to poor sensitivity.

One Step Further Question: How frequently do patients with testicular torsion report a history of similar pain?

Answer: Up to 29% of patients with testicular torsion will report similar pain in the past.

One Step Further Question: What is the cause of struvite renal stones?

Answer: Urease producing bacteria like Proteus, Klebsiella and Staphylococcus.

One Step Further Question: Can testicular torsion occur during sleep?

Answer: Yes. Up to 50% of all cases are reported to occur during sleep.

A 54-year-old man presents to the ED in acute renal failure. Which of the following findings on urinalysis is most suggestive of acute tubular necrosis as the cause for this patient's acute renal failure? Brown granular casts Hyaline casts Red cell casts White cell casts

Correct Answer ( A ) Explanation: Acute tubular necrosis (ATN) is a severe form of impairment of tubular epithelial cells caused by ischemia or toxic injury. It is a leading cause of ARF. One of its hallmarks is the presence of brown granular casts on urinalysis. These contain cellular debris rich in cytochrome pigments. In contrast, hyaline casts (B) are usually nonspecific but present after exercise; red cell casts (C) are indicative of glomerular hematuria (e.g., glomerulonephritis); and white cell casts (D) imply renal parenchymal inflammation (e.g., acute interstitial nephritis, pyelonephritis).

Which of the following infectious agents is associated with reactive arthritis? AAdenovirus BChlamydia trachomatis CStaphylococcus aureus DStreptococcus pyogenes

Correct Answer ( B ) Explanation: The development of reactive arthritis is associated with a number of bacterial agents including Chlamydia trachomatis. Reactive arthritis is characterized by oligoarthritis, urethritis and conjunctivitis/uveitis - "can't see, can't pee, can't climb a tree." The syndrome occurs in susceptible hosts after infection with certain agents including C. trachomatis. The arthritis typically develops 2-6 weeks after an episode of urethritis or dysentery. The most commonly involved joints are the ankles and knees and the syndrome typically involves bilateral joints. Conjunctivitis may be present early and progress to uveitis later in the disease process. Management focuses on decreased inflammation with non-steroidal anti-inflammatory medications. C. trachomatis-triggered reactive arthritis improves faster with antibiotics.

A 73-year-old man with benign prostatic hyperplasia presents with a failure to void for 3 days and lower abdominal pain. Physical examination reveals lower abdominal distension and moderate tenderness to palpation. His BUN/Creatinine is 20/1.6. What management is indicated? Emergency hemodialysis Foley catheter placement Intravenous fluids Suprapubic catheterization

Correct Answer ( B ) Explanation: The patient presents with symptoms and signs consistent with acute urinary obstruction requiring placement of a Foley catheter to relieve the obstruction. Postrenal or obstructive renal failure is a reversible cause of acute kidney injury (AKI) and should always be considered in patients with new azotemia. Benign prostatic hyperplasia (BPH) is a common cause of urinary obstruction. Treatment of obstructive uropathy focuses on relieving the obstruction. There is no contraindication to placing a Foley catheter in patients with BPH and it should be done emergently to relieve obstruction. Emergency dialysis (A) should only be undertaken for pulmonary edema, severe uncontrollable hypertension, hyperkalemia, severe electrolyte or acid-base disturbance, specific overdoses and severe, symptomatic uremia. Intravenous fluids (C) to restore circulating volume are not necessary in obstructive renal failure. Suprapubic catheterization (D) should be attempted if placement of a Foley catheter fails.

Which of the following is the most common cause of bladder calculi? Chronic indwelling catheter Hyperparathyroidism Infection of residual bladder urine with urea-splitting organisms Prostatitis

Correct Answer ( C ) Explanation: Bladder stones are much less common than renal or ureteral calculi. In the United States, bladder stones occur almost exclusively in elderly men, often as a complication of other urologic disease. They are usually associated with urinary stasis but can form in healthy individuals without evidence of anatomic defects, strictures, infections, or foreign bodies. The most common cause is infection of residual bladder urine with urea-splitting organisms. Disorders predisposing to the formation of bladder stones include bladder neck obstruction, neurogenic bladder, vesical diverticula, damage from irradiation, and schistosomiasis. Most vesical calculi are formed de novo within the bladder. Renal stones small enough to pass through the ureters are also small enough to pass through a normally functioning bladder and an unobstructed urethra. Clinically, patients with bladder stones most often complain of pain on voiding and hematuria. A classic complaint is the sudden interruption of the urinary stream. The physical exam is usually unremarkable. CT scan is the most sensitive diagnostic study to identify bladder stones. Because a bladder stone is in itself a sign of an underlying problem, removal of the stone and treatment of the underlying abnormality are nearly always indicated. A chronic indwelling catheter (A) is also a common cause of bladder stones, but not the most common. Hyperparathyroidism (B) leads to increased serum levels of calcium and is a predisposing factor to renal stones. However, it is not the most common cause of bladder stones. Prostatitis (D) does not increase the risk of bladder stones. However, prostatic hypertrophy can lead to urinary obstruction and cystitis, which predispose to bladder calculi.

A 45-year-old man presents to the emergency department complaining of sharp left sided back pain. The patient states the pain began suddenly, waxes and wanes in intensity, and worsens with urination. He also complains of nausea and red tinged urine. On examination, he is afebrile and pacing the room. There is left costovertebral angle tenderness on palpation with a soft, non-tender abdomen. You suspect nephrolithiasis. Which imaging modality is most sensitive and specific in confirming this diagnosis? Abdominal ultrasound Intravenous pyleography Non-contrast helical computed tomography scan Plain radiograph of kidneys, ureter, and bladder

Correct Answer ( C ) Explanation: Nephrolithiasis, a kidney stone, it forms when calcium oxalate, struvite, cysteine, or uric acid build up in the urine resulting in crystal formation within the kidney. Calcium oxalate stones are the most common. The prevalence of kidney stones is approximately 2-3% of the general population, males being more affected than females. Risk factors for developing kidney stones include decreased fluid intake, concentrated urine, dietary habits, and certain medications and diseases. Pain in the most common presenting symptom, which occurs as the stone, descends the ureter, obstructing urine output and causing inflammation and swelling of the ureter. While the stone is in the kidney it typically does not cause pain. The pain is located to the unilateral mid-back or flank region and low abdomen, and often radiates to the ipsilateral groin particularly during urination. The pain does not improve with positional changes. Other symptoms can include hematuria, nausea, vomiting, and dysuria. The diagnosis of kidney stones is often based primarily on history, but imaging is important to exclude other potential causes of abdominal pain such as appendicitis, aortic aneurysm, or pyelonephritis. A non-contrast helical computed tomography (CT) scan is the most sensitive (95-100%) and specific (94-96%) imaging modality in confirming the presence of a kidney stone. A CT scan is also fast, readily available in an emergent situation, definitive, and identifies other signs of ureteral obstruction (e.g. ureter distension or perinephric stranding) as well as other nonurological conditions that may also cause abdominal pain.

Which of the following is characteristic of nephrotic syndrome? Anemia Hyperglycemia Hypoalbuminemia Low urinary protein output

Correct Answer ( C ) Explanation: Nephrotic syndrome is characterized by high urine protein excretion (>3.5 g/24 hr), peripheral edema, and metabolic abnormalities (hypoalbuminemia, hypercholesterolemia). The pediatric definition of nephrotic syndrome is proteinuria greater than 50 mg/kg/day or a urine protein/creatinine ratio of more than 2.0 mg/mg; hypoalbuminemia of less than 2.5 g/dL; and the presence of edema and hyperlipidemia. The pathogenesis of nephrotic syndrome has long been investigated with an emphasis on the mechanisms of glomerular injury and proteinuria. Nephrotic syndrome in children is traditionally classified into three categories: primary (also designated idiopathic nephrotic syndrome), secondary, and congenital nephrotic syndrome. Idiopathic nephrotic syndrome is subcategorized according to histopathologic appearance, mainly including minimal change disease (MCD), focal segmental glomerulosclerosis (FSGS), membranoproliferative glomerulonephritis (MPGN), membranous nephropathy, and diffuse mesangial proliferation. The term secondary nephrotic syndrome refers to nephrotic syndrome associated with an identifiable systemic disease or infection and is often accompanied by a nephritic component. Congenital nephrotic syndrome has been defined as nephrotic syndrome occurring within the first 3 months of life. Neither anemia (A) or hyperglycemia (B) are associated with nephrotic syndrome. Patients with nephrotic syndrome have high urinary output of protein, not low urinary output (D).

9-year-old boy presents with an inability to retract his foreskin. He has not been able to urinate for two days. The visible portion of the glans appears dusky. Which of the following therapies is most appropriate? Circumcision Corticosteroid cream Dorsal slit procedure Intracavernosum phenylephrine

Correct Answer ( C ) Explanation: This patient presents with phimosis and evidence of vascular compromise requiring performance of a dorsal slit procedure. Phimosis occurs when the foreskin of an uncircumcised penis becomes constricted preventing retraction of the prepuce from the glans. It results in urinary outlet obstruction, glans ischemia and infarction. Many cases are physiologic resulting from normal development. Patients typically present with an unretractable foreskin and symptoms consistent with urinary obstruction including decreased urinary stream. Management of phimosis can be difficult. Dilation of the prepuce can be performed using forceps but often does not result in relief of phimosis. When there are signs of glans vascular compromise (i.e. discoloration) a dorsal split procedure should be performed. During this procedure, the foreskin is anesthetized and incised dorsally allowing for retraction. A circumcision (A) may be necessary for definitive care but treatment should not be delayed. Patients with no signs of obstruction or vascular compromise can be treated with topical corticosteroids (B) for 6 weeks but require close follow up. Intracavernosum phenylephrine (D) is a treatment modality for priapism but plays no role in the treatment of phimosis.A circumcision (A) may be necessary for definitive care but treatment should not be delayed. Patients with no signs of obstruction or vascular compromise can be treated with topical corticosteroids (B) for 6 weeks but require close follow up. Intracavernosum phenylephrine (D) is a treatment modality for priapism but plays no role in the treatment of phimosis.

Which of following statements regarding nitrofurantoin is correct? It can be safely prescribed to neonates for treatment of urinary tract infections It can be safely prescribed to patients with G6PD deficiency It is considered effective therapy for treatment of pyelonephritis It is considered effective therapy for treatment of uncomplicated cystitis

Correct Answer ( D ) Explanation: Nitrofurantoin is bactericidal and interferes with cell wall synthesis. The exact mechanism of action is poorly understood. It is effective for the treatment of uncomplicated lower urinary tract infections caused by E. coli, S. saprophyticus, and E. faecalis. Although effective in the lower urinary tract, nitrofurantoin has poor tissue penetration and low serum levels. Therefore it should not be used to treat pyelonephritis (C) or prostatitis. It is considered pregnancy category B, however is contraindicated in the third trimester and neonatal period (A), as neonatal immature red blood cells are damaged by nitrofurantoin causing hemolytic anemia. Nitrofurantoin can also cause hemolytic anemia in individuals with G6PD deficiency (B).

A 23-year-old man presents with testicular pain for 3 hours. He denies any trauma. The pain is constant, sharp and severe and is accompanied by nausea and vomiting. His vital signs are normal except for a heart rate of 110 beats/min. On examination, there is no swelling but the left testicle is extremely tender to palpation. The cremasteric reflex is absent. What management is indicated? Ceftriaxone 250 mg intramuscular X 1 and doxycycline 100 mg BID X 10 days CT scan of the abdomen and pelvis without contrast Immediate scrotal ultrasound followed by urology consultation Immediate urology consultation followed by scrotal ultrasound

Correct Answer ( D ) Explanation: This patient has signs and symptoms concerning for testicular torsion; one of the few true urologic emergencies. Torsion of the testes results from an extravaginal or intravaginal defect leading to twisting of the spermatic cord and resultant ischemia to the testicle. Although the majority of torsion occurs in the absence of trauma, MVC and straddle injuries have been associated with this disorder. Initially, twisting causes decreased venous return but eventually, arterial obstruction occurs. Physical exam often reveals a tender swollen testicle with a transverse lie that may be higher than the unaffected testicle due to shortening of the spermatic cord with twisting. An absent cremasteric reflex (elevation of the testicle caused by stroking of the inner thigh) is nearly universal (100% in patients >30 months of age). The duration of vascular obstruction affects the testicular salvage rate. Torsion recognized within 6 hours of symptom onset is associated with a salvage rate of 80-100% while delay to 24 hours or longer is almost always associated with a loss of the testicle. Because of the time sensitive nature of the disorder, urologic consultation must be immediately obtained as the only definitive way to diagnose and treat testicular torsion is with exploration and detorsion in the operating room. Manual detorsion can be attempted in the Emergency Department but is only successful 26% of the time. A scrotal ultrasound (C) can be obtained in patients with an equivocal history or physical examination but should not delay surgical consultation in patients with a high clinical suspicion. Sensitivity of ultrasound varies from 88-100% and thus, in a high-risk patient with a negative ultrasound, surgical exploration should still be considered. A CT scan of the abdomen and pelvis (B) without contrast is useful in looking for renal colic which can produce sudden, severe pain with referred pain to the testicle. However, testicular tenderness and an absent cremasteric reflex are not seen in renal colic. Ceftriaxone and doxycycline (A) are indicated in the treatment of epididymitis. Epididymitis can present with generalized testicular pain but it is usually gradual in onset and the cremasteric reflex should be intact.

A 72-year-old man is brought to the ED from a nursing home for evaluation of oliguria. He is found to have an acutely elevated BUN and plasma creatinine from baseline. A Foley catheter is placed; his urine sodium (UNa) is measured below 20 mEq/L and fractional excretion of sodium (FENa) below 1%. Which of the following is most consistent with these findings? Acute tubular necrosis Loop diuretic medication Osmotic diuresis Prerenal azotemia

Correct Answer ( D ) Explanation: This patient's oliguria with acutely elevated BUN and plasma creatinine suggest that he is in acute renal failure (ARF) or Acute kindey injury (AKI). His UNa <20 mEq/L and FENa <1% indicate that he has intact reabsorptive function and is able to conserve sodium. This is consistent with prerenal azotemia as the cause for his ARF. Acute tubular necrosis (ATN) (A), loop diuretics (e.g., furosemide) (B), and osmotic diuresis (e.g., mannitol) (C) all lead to UNa >20 mEq/L and FENa >1% because there is impairment in the ability to concentrate the urine. In such cases, a high-sodium load is excreted.

A 68-year-old man with chronic kidney disease is sent to the ED following hemodialysis due to bleeding from the dialysis puncture site of his arteriovenous fistula (AVF). An emergency physician applies local pressure for several minutes, and the bleeding stops. The patient then notices that the thrill over the puncture site is gone. Which of the following is the best next step in management? Irrigation with normal saline Manual manipulation Thrombolytic administration (locally) Vascular surgery consultation

Correct Answer ( D ) Explanation: When applying local pressure to a bleeding puncture site, care must be taken to not occlude the AVF because this can cause thrombosis. For this reason, it is important to check for and document the presence of a thrill after applying pressure. In the event that a suspected thrombotic event has occurred, it is pertinent to consult a vascular surgeon immediately because definitive treatment often involves surgical revision. The access device should not be irrigated (A) or manipulated (B) because this may cause venous embolization or vessel rupture. Thrombolytic agents (C) are sometimes used; however, surgical intervention is often warranted, and a discussion should take place with a vascular surgeon before a decision is made to administer thrombolytics.

Rapid Review Cystitis

Cystitis Patient will be complaining of low-grade fever, increased urinary frequency, dysuria, and suprapubic or abdominal pain. Labs will show positive leukocyte esterase and nitrites Definitive diagnosis is made by urine culture Most commonly caused by Escherichia coli Treatment is: Acute uncomplicated cystitis: TMP-SMX, nitrofurantoin, or fluoroquinolone for 3-5 days Acute uncomplicated cystitis with comorbid conditions: TMP-SMX, nitrofurantoin, or fluoroquinolone for 7 days Pregnancy - asymptomatic bacteruria - treat Cephalosporins, nitrofurantoin (during first trimester - only when needed) Complications: ↑ Risk of preterm birth, low birth weight, perinatal mortality

One Step Further Question: Name 5 urea-splitting bacteria?

One Step Further Question: Name 5 urea-splitting bacteria?

One Step Further Question: True or false: Asymptomatic bacteriuria in pregnancy should be treated with antibiotics.

Answer: True.

One Step Further Question: What are the most common etiologic agents responsible for urinary tract infections?

Answer: Gram-negative aerobic bacilli that arise from the GI tract, such as E. coli, account for approximately 80% of cases.

One Step Further Question: What constitutes a positive urine culture in a catheterized specimen?

Answer: Growth of >100,000 colony forming units (CFUs).

One Step Further Question: What kind of casts would be expected on a urinalysis in a patient with prerenal azotemia?

Answer: Hyaline casts or no casts at all.

One Step Further Question: What is seen on electron microscopy of the glomerulus in patients with poststreptococcal glomerulonephritis?

Answer: Immune deposits in the subepithelial space.

One Step Further Question: What condition should be considered in individuals diagnosed with a right sided varicocele?

Answer: Inferior vena cava thrombosis.

One Step Further Question: What is balanoposthitis?

Answer: Inflammation of the glans and foreskin.

One Step Further Question: What is the reaction that can occur during the treatment of syphilis that causes fever, chills, rash, and myalgias?

Answer: Jarisch-Herxheimer reaction.

One Step Further Question: What is the most common complication of PD after infection?

Answer: Leakage of fluid into surrounding tissue (particularly into the scrotum in men).

One Step Further Question: Which side are varicoceles more common?

Answer: Left side.

One Step Further Question: Why is nitrofurantoin not used in the treatment of pyelonephritis?

Answer: Nitrofurantoin is excreted unchanged in the urine only reaching effective concentrations in the bladder.

One Step Further Question: Does the presence of Prehn's sign rule out testicular torsion?

Answer: No.

One Step Further Question: What are the most common causes of obstructive urinary retention in women?

Answer: Pelvic masses and prolapse of the pelvic organs (e.g. bladder, rectum, or uterus).

One Step Further Question: For patients who develop DS, what is an alternative to HD?

Answer: Peritoneal dialysis, with which DS has not been reported.

One Step Further Question: What is the name of the inability to retract foreskin?

Answer: Phimosis.

One Step Further Question: What medications are commonly associated with priapism?

Answer: Phosphodiesterase 5 inhibitors (eg sildenafil), trazadone, papaverine and phenothiazines

One Step Further Question: What are the most common causes of acute urinary obstruction in women?

Answer: Prolapse of pelvic organs and pelvic tumors.

One Step Further Question: What diagnostic test should be performed on this patient during his ED evaluation?

Answer: Scrotal ultrasound.

One Step Further Question: Which pregnant patients with bacteriuria should be admitted for intravenous antibiotics?

Answer: Those patients in their last trimester, ill appearance or evidence of pyelonephritis.

One Step Further Question: Which patients with suspected renal stone require imaging?

Answer: First episode of renal colic, unclear diagnosis, concomitant UTI, the elderly.

One Step Further Question: When should a patient with prostatitis be referred to a urologist?

Answer: Toxic patients, complicated infections, chronic cases and those that don't resolve with standard therapy.

One Step Further Question: When considering scrotal pain, which signs favor a diagnosis of testicular torsion over epididymitis?

Answer: Acute severe pain, high-riding testicle, pain worsens with testicle elevation, absent cremasteric reflex, normal testicle but decreased blood flow on color Doppler ultrasonography.

One Step Further Question: What is the most common antibiotic linked to acute tubular necrosis?

Answer: Aminoglycosides.

One Step Further Question: What are 2 serious illnesses that mimic renal colic?

Answer: Aortic/iliac aneurysm and renal vein thrombosis.

One Step Further Question: What sign on physical examination is associated with torsion of the testicular appendage?

Answer: Blue dot sign.

One Step Further Question: What is the definitive treatment for paraphimosis?

Answer: Circumcision.

One Step Further Question: What is the definitive treatment of paraphimosis?

Answer: Circumcision.

One Step Further Question: What is a cause of recurrent bleeding from a dialysis puncture site that requires surgical evaluation?

Answer: Development of an aneurysm or pseudoaneurysm.

One Step Further Question: Which therapy offers definitive management in hyperkalemia?

Answer: Dialysis.

One Step Further Question: What is the late complication of rhabdomyolysis associated with thrombocytopenia, hypofibrinogenemia, and an elevated D-dimer?

Answer: Disseminated intravascular coagulopathy may occur and is a result of muscle necrosis with liberation of activating substances from injured cells.

One Step Further Question: In paraphimosis, if manual reduction of the foreskin cannot be achieved, what is the next step?

Answer: Dorsal slit procedure.

One Step Further Question: What is the most common organism that causes acute pyelonephritis?

Answer: Escherichia coli.

One Step Further Question: Why are patients with Crohn's disease at risk for nephrolithiasis?

Answer: Fat malabsorption is common in Crohn's disease, leading to fat accumulation in the gut. Fat binds calcium leaving oxalate free to be absorbed and deposited in the kidney, where it can lead to stone formation.

Which of the following is characteristic of testicular torsion? Associated with a loss of the cremasteric reflex Common complication of cystoscopy Elevating the testicle relieves the pain Increased incidence in patients with a history of sexually transmitted infection

Correct Answer ( A ) Explanation: Although occasionally seen in the newborn male, testicular torsion is an acquired condition seen most commonly in infants, teens, and men <30 years of age. It usually presents with abrupt onset of severe scrotal pain that may wake the patient up from sleep. Associated signs and symptoms include fever, nausea, vomiting, and abdominal pain. On physical examination, the affected testicle is tender and swollen. The cremasteric reflex is invariably absent. The reflex is induced by stroking the inner thigh. The normal response is a contraction of the cremasteric muscle which pulls the ipsilateral testis upwards. The testis may lie in a more horizontal position caused by a lack of normal attachment to the tunica vaginalis ("bell clapper" deformity). If the diagnosis is suspected, immediate consultation with a urologist is warranted. Ultrasound can aid the diagnosis, but can be falsely negative. Timeliness of the diagnosis is critical because testicular viability rapidly declines with time. A history of sexually transmitted infections (D) does not increase the likelihood for testicular torsion. Epididymitis is a complication of cystoscopy (B), not torsion. Elevating the testicle (C) usually induces pain, rather than relieves pain. In some cases, testicular elevation relieves pain in epididymitis (Prehn's sign).

A 45-year old man presents to the ED with a complaint of left flank pain, nausea and vomiting. A non-contrast CT scan of the abdomen and pelvis confirms the diagnosis of a renal calculus. Which of the following factors would necessitate hospital admission? CT scan shows a 4 mm calculus with associated mild hydronephrosis and urinalysis shows evidence of a urinary tract infection CT scan shows a 4 mm calculus without associated hydronephrosis and urinalysis shows evidence of urinary tract infection CT scan shows a 6 mm calculus with associated mild hydronephrosis and urinalysis shows no evidence of urinary tract infection CT scan shows a 6 mm calculus without associated hydronephrosis and urinalysis shows no evidence of urinary tract infection

Correct Answer ( A ) Explanation: Concomitant obstruction and infection is an absolute indication for hospital admission and should prompt emergency consultation with urology. The decision to admit or discharge a patient with renal calculi is based on the clinical scenario, as well as the laboratory results and imaging studies. Absolute indications for hospital admission include: concomitant obstruction and infection, intractable vomiting, uncontrollable pain, urinary extravasation and hypercalcemic crisis. Relative indications for admission include: solitary kidney, intrinsic renal disease, significant comorbid conditions (e.g. immunosuppression), high-grade obstruction, leukocytosis and psychosocial factors affecting ability to follow-up or be managed in the outpatient setting. Stones less than 5 mm in size pass spontaneously 90% of the time. Stones 5-8 mm in size pass 15% of the time, those that are 4-6 mm pass 50% of the time. Stones greater than 8 mm become impacted 95% of the time. Stone size (D) alone does not necessitate admission particularly when there is no associated obstruction or infection. Generally patients with mild to moderate hydronephrosis can safely be managed in the outpatient setting, as long as there is no concurrent infection (C). Patients with mild infections can also be managed in the outpatient setting as long as there is no concurrent obstruction (B).

A 46-year-old man with a history of chronic kidney disease is brought to the ED from the hemodialysis unit in the hospital due to the development of a headache, nausea, and general malaise while undergoing hemodialysis. He felt well prior to the initiation of hemodialysis. Currently, he denies fevers, chills, coughing, or chest pain. His vital signs are T 36.4°C, BP 114/82, HR 86, RR 15, SpO2 99% on room air. He is alert and oriented though he appears uncomfortable and nauseous. His cardiorespiratory and neurologic exams are within normal limits. Which of the following is the most likely diagnosis? Disequilibrium syndrome Meningitis Subdural hematoma Uremia

Correct Answer ( A ) Explanation: Disequilibrium syndrome (DS) is characterized by headache, malaise, nausea, disorientation, and—in severe but rare cases—confusion, seizures, coma, and death. It occurs during hemodialysis (HD) or shortly after. By most accounts, it is attributed to the rapid removal of urea during HD, leading to transient cerebral edema. Treatment is largely supportive in addition to ruling out more serious etiologies of the symptoms. Meningitis (B) is less likely than DS, given this patient's symptom onset during HD, his normal mental status, and lack of fever or neck pain. Any form of intracranial hemorrhage (C) is in the differential, although less likely, and neuroimaging would more likely be prompted by a change in this patient's mental status or the presence of focal neurologic deficits. Uremia (D) would be more likely to improve than worsen during HD, although severe uremia prior to the onset of HD increases one's risk for the development of DS.

26-year old sexually active man presents with a 3-day history of unilateral, painful testicular swelling. He reports subjective fevers and dysuria and denies nausea and vomiting. Urinalysis shows leukocyte esterase and greater than 10 white blood cells. What is the next step in management for this patient? Doxycycline and ceftriaxone therapy Levofloxacin Testicular ultrasound Urethral swab and Gram stain

Correct Answer ( A ) Explanation: Epididymitis is the most common cause of scrotal pain in adults and is characterized by acute unilateral pain and swelling. The pain usually begins at the epididymis and can spread to the entire testicle (epididymo-orchitis). Other symptoms include fever, erythema of the scrotal skin, and dysuria. It is associated with a C-reactive protein level greater than 24 mg per L and increased blood flow on ultrasonography. Chlamydia trachomatis and Neisseria gonorrhoeae are the most common organisms responsible for bacterial epididymitis in males younger than 35 years. Guidelines recommend ceftriaxone (250 mg IM x 1) and doxycycline (100 mg PO BID x 10 days) for treatment of suspected epididymitis in males younger than 35 years. Levofloxacin (B) is the treatment of choice for epididymitis-orchitis in males older than 35 years of age. The most common pathogens in this population are the enteric bacteria (e.g. E. coli). A testicular ultrasound (C) is indicated if there is any concern for testicular torsion, a surgical emergency that requires immediate consultation with a urologist. Characteristics that suggest testicular torsion include rapid symptom onset, nausea and vomiting, high position of the testicle, and absent cremasteric reflex. Surgical repair must occur within six hours of symptom onset to reliably salvage the testicle. Urethral swab and Gram stain (D) is typically reserved for patients with suspected acute urethritis and not epididymitis.

A 19-year-old boy presents with scrotal pain and fever. Examination reveals a tender, swollen testicle. Scrotal Doppler ultrasonography shows increased blood flow to the testicle. Elevation of the scrotum lessens the patient's pain. Which of the following organisms is the most common cause of this condition in this patient? Chlamydia trachomatis Escherichia coli Group A Streptococcus Staphylococcus epidermidis

Correct Answer ( A ) Explanation: Epididymitis occurs most commonly in men between the ages of 14 and 35 years. Epididymitis is characterized by a gradual onset of scrotal pain, fever and urinary urgency, frequency, dysuria, pyuria or hematuria. Examination usually reveals localized epididymal edema and tenderness (posterior aspect of scrotum), possible testicular tenderness, and a normal cremasteric reflex. Pain may be relieved with testicular elevation (positive Prehn sign). Scrotal pain should be initially evaluated with a color Doppler ultrasound test, and in the case of epididymitis, the typical findings are an enlarged, thickened epididymis with increased blood flow. The most common organism responsible for epididymitis in those 14 to 35 years-of-age are Neisseria gonorrhoeae and Chlamydia trachomatis. In older individuals (traditionally > 35 years of age), the gram-negative rod bacteria (Escherichia, Klebsiella, Enterobacter and Citrobacter species) are most common. Escherichia coli (B) is more common in men with epididymitis over the age of 35 years. Group A Streptococcus (C) is not associated with deep genitourinary infections. It is however a common cause of superficial epidermal conditions such as cellulitis, impetigo, erysipelas and scarlet fever. Staphylococcus epidermidis (D) is a more common cause of urinary tract infection secondary to external devices, such as a urinary catheter.

A 61-year-old previously healthy man presents with three days of gross hematuria. He denies any dysuria, flank pain, fever, or trauma. On examination, his heart rate is 78 beats per minute, blood pressure 142/72 mm Hg, and temperature 37.4°C. He has no abdominal tenderness or fullness on palpation. Genitourinary exam is unremarkable. Urinalysis is negative for leukocyte esterase and nitrites. There are > 100 red blood cells/hpf and 6 white blood cells/hpf. Creatinine is 1.1. What is the next step in the management of this patient? Computed tomography of the abdomen and pelvis with intravenous contrast Continuous bladder irrigation Discharge home with ciprofloxacin for five days Discharge home with urology referral

Correct Answer ( A ) Explanation: Hematuria is a frequently encountered condition in the emergency department, whether a patient presenting with gross hematuria or microscopic hematuria is found incidentally on urinalysis. Bleeding anywhere along the upper or lower genitourinary tract can produce hematuria, with the most common causes being urinary tract infection, trauma, and renal calculi. Unlike microscopic hematuria, which is often transient and not indicative of serious underlying pathology, gross hematuria (especially in older patients) is often a presenting symptom of malignancy. Risk factors that should prompt a thorough evaluation include age > 40 years, history of cigarette smoking, persistent hematuria, abdominal pain, and trauma. Workup should include a full history and physical, urinalysis, and assessment of renal function. If findings do not point to a specific cause of hematuria, a CT scan with IV contrast or renal ultrasound should be ordered to rule out underlying structural disease or mass. Continuous bladder irrigation (A) is indicated in patients with active bleeding that results in urinary retention due to obstruction with clots. Discharge home with ciprofloxacin (C) would be an appropriate treatment for a patient with an uncomplicated urinary tract infection. This patient is not exhibiting symptoms consistent with an infection (e.g. dysuria, urgency, frequency, fever) and his urinalysis is also negative for infection. The positive predictive value for malignancy in patients over age 60 with gross hematuria is over 20% in men and 8% in women. Discharge home with urology referral (D) prior to imaging would not be appropriate in this patient population.

A 65-year-old man presents with acute urinary retention. What is the most common cause of acute urinary retention in this patient? Benign prostatic hyperplasia Urethral stricture Urinary calculus Urinary tract infection

Correct Answer ( A ) Explanation: In men over the age of 50 years of age, benign prostatic hyperplasia (BPH) is the most common cause of acute urinary retention. Acute urinary retention is the sudden inability to void urine voluntarily. It is most commonly caused by obstructive lesions. BPH is the most common precipitant and is implicated in about 50% of patients. Urinary output is obstructed by enlargement of the prostate and constriction of the prostatic urethra (from alpha-adrenergic tone). Approximately 25% of men with acute urinary retention will have prostate cancer instead of BPH. A urinary calculus (C) in the urethra can cause acute urinary retention. Urinary tract infections (D) are less common in men and rarely causes them to become obstructed. Other less common causes include urethral strictures (B) and phimosis.

Which of the following patients with a renal stone should be considered for admission? A 24-year-old man with a 5 mm stone at the ureterovesicular junction who presents with intractable nausea and vomiting A 33-year-old pregnant woman with a 2 mm stone at the ureteropelvic junction who is tolerating oral intake and has 2+ hematuria A 37-year-old man with a 3 mm stone at the pelvic brim; ultrasound shows no hydronephrosis; urinalysis shows 2+ blood and 5-10 WBCs A 45-year-old man with a 7 mm renal stone at the ureterovesicular junction who required a single dose of pain medication to resolve his pain

Correct Answer ( A ) Explanation: Most patients with renal stones may be safely discharged home from the ED. The patient in the above scenario, with intractable nausea and vomiting, should be admitted to the hospital for symptomatic treatment relief and fluid resuscitation. Sepsis and renal damage are risks in the presence of obstruction and infection. These patients require immediate urologic consultation to evaluate the need for drainage and for relief of the obstruction by ureteral stenting. If infection is present, patients require fluid resuscitation and antibiotics. A 33-year-old pregnant woman with a 2 mm stone at the ureteropelvic junction who is tolerating oral intake and has 2+ hematuria (B) can be discharged, but will require close follow-up. There should be a low threshold to admit pregnant women with renal stones, particularly when there are signs of infection or dehydration. A 37-year-old man with a 3 mm stone at the pelvic brim, where ultrasound shows no hydronephrosis and urinalysis shows 2+ blood and 5-10 WBCs (C) can be discharged. His stone is likely to pass spontaneously given its size. Although he has WBCs in his urine, there is no evidence of obstruction seen by the lack of hydronephrosis on ultrasound. A 45-year-old man with a 7 mm renal stone at the ureterovesicular junction who required a single dose of pain medication to resolve his pain (D) can be discharged with outpatient follow-up to a urologist. His stone is less likely to pass spontaneously. Therefore, he may require lithotripsy. This can be done outpatient.

An 18-year-old woman presents to the ED with dark-colored urine and malaise for the past three days. Her vital signs are BP 155/85 mm Hg, HR 80, RR 16, and T 36.7°C. On exam, you note 1+ pretibial edema. Urinalysis reveals proteinuria, hematuria, and red blood cell casts. Which of the following is the most likely diagnosis? Acute glomerulonephritis Acute tubular necrosis Hypothyroidism Nephrotic syndrome

Correct Answer ( A ) Explanation: Patients with acute glomerulonephritis present with a spectrum of clinical signs and symptoms. The presence of hematuria, proteinuria, and red blood cell casts are highly suggestive of the diagnosis. In fact, the presence of red blood cell casts alone is highly specific for acute glomerulonephritis. Conversely, if hematuria, proteinuria, and red blood cell casts are absent, glomerulonephritis is highly unlikely. In addition to the urinary findings, patients may present with hypertension, edema, or congestive heart failure secondary to volume overload. Acute tubular necrosis (B) is an intrinsic cause of acute kidney injury that is associated with a variety of renal insults, including ischemia and nephrotoxins. Muddy brown casts are often seen on urinalysis. Hypothyroidism (C) is associated with fatigue, weakness, edema, and urinary retention, but there are no specific findings on urinalysis. Nephrotic syndrome (D) causes edema and proteinuria but is not associated with hematuria or red blood cell casts.

An 18-year-old boy presents to the emergency department with severe pain in his penis. He indicates he has had some mild irritation to the glans and foreskin areas recently. After intercourse tonight he was unable to reduce the foreskin and now has severe pain in the tip of his penis. On examination, the penis is flaccid and the foreskin is retracted proximally. There is swelling to the tip of the penis with mild erythema. What is the most likely diagnosis? Paraphimosis Penile fracture Phimosis Priapism

Correct Answer ( A ) Explanation: The patient in this question has a story and exam consistent with paraphimosis. Paraphimosis is a true urologic emergency of the uncircumcised penis. It is the inability to reduce the proximal edematous foreskin distally over the glans penis into its natural position. The resulting glans edema and venous engorgement can progress to arterial compromise and gangrene. Paraphimosis can be caused by infection, masturbation, trauma, or hair or clothing tourniquets. Patients typically appear very anxious and the history reveals that the patient retracted the foreskin and could not replace the foreskin over the glans. Physical examination reveals a flaccid proximal penis with erythema and engorgement distal to the obstruction. The foreskin is retracted and cellulitis may be present. The diagnosis of paraphimosis is based on clinical findings and should be intervened on immediately. Paraphimosis can often be reduced by compression of the glans for several minutes to reduce edema and allow for successful reduction of the now smaller glans through the foreskin. A penile fracture (B) occurs when the tunica albuginea of one or both corpora cavernosa ruptures due to direct trauma to the erect penis. The most common cause is sexual intercourse, but other causes include animal bites, stabbing, bullet wounds, and self-mutilation. On examination, the penis is acutely swollen but flaccid, discolored, and tender. Phimosis (C) is the inability to retract the foreskin proximally. Infection, poor hygiene, and previous preputial injuries with scarring are common causes of pathologic phimosis. Scarring at the tip of the foreskin can occlude the preputial meatus, infrequently causing urinary retention. Priapism (D) is a urologic emergency that is characterized by a persistent, painful, pathologic erection in which both corpora cavernosa are engorged with stagnant blood. Many cases of priapism are related to oral antihypertensive agents, neuroleptic medications, or oral agents related to erectile dysfunction.

A 32-year-old woman presents to the emergency department with dysuria, back pain, and fever for two days. She indicates she has had urinary tract infections in the past but never this severe. She denies other past medical history and does not take any medications. Vital signs show HR 114, BP 132/80, R 16, T 102.2°F. Examination shows a moderately ill-appearing woman with right costovertebral angle tenderness. What test or tests are required in making an appropriate diagnosis in this patient? Urinalysis Urinalysis and contrast enhanced computed tomography of abdomen/pelvis Urinalysis and non-contrast enhanced computed tomography of abdomen/pelvis Urinalysis and ultrasonography of renal parenchyma

Correct Answer ( A ) Explanation: The patient in this question is presenting with signs and symptoms of acute pyelonephritis. Acute pyelonephritis is a urinary tract infection of the renal parenchyma and collecting system manifesting with the clinical syndrome of fever, chills, flank pain, and nausea or vomiting in conjunction with a urinalysis that is consistent with infection. The majority of patients with acute pyelonephritis do not need emergency imaging to secure the diagnosis of pyelonephritis. Patients with either unusually severe signs and symptoms or an atypical presentation are candidates for imaging. Additionally, patients with a known current history of urinary tract infection who are receiving antibiotics but persistently have fevers and general toxicity require further imaging to look for complications from pyelonephritis such as abscess formation. A contrast-enhanced computed tomography of abdomen/pelvis (B) is perhaps the best test for assessing the kidneys, ureters, and bladder. It has the highest sensitivity for detecting abscess, obstruction, and acute inflammation. Its disadvantages include cost, radiation exposure, and time to complete. In this otherwise healthy patient with classic symptoms, the test would not be necessary. A non-contrast enhanced computed tomography of abdomen/pelvis (C) is preferred in patients with potential ureteral obstruction from a renal stone, but it is not the preferred study to evaluate for inflammatory changes. Urinalysis and ultrasonography (D) are useful in the evaluation of patients with potential urinary obstruction. Ultrasound is a sensitive tool for detecting intrarenal and perinephric abscess and the presence of hydroureter and hydronephrosis but is less accurate in determining the presence of a partially obstructing ureteral stone. It can detect pyelonephritis but it is not necessary in this otherwise healthy and uncomplicated patient.

A 51-year-old woman presents with flank pain and fever. CT scan shows an obstructing 4 mm stone at the uretrovesicular junction. Urinalysis reveals 25-50 WBCs. What management is indicated? Antibiotics and inpatient urology consultation Antibiotics and outpatient follow up with urology Intravenous hydration and follow up with urology Pain control and follow up with urology

Correct Answer ( A ) Explanation: The patient presents with an obstructing stone and evidence of a urinary tract infection requiring inpatient management and urology consultation. Urolithiasis affects 5-15% of the world's population. The majority of stones are made of calcium oxalate with or without calcium phosphate. Ureteral obstruction from urolithiasis can cause impaired renal function and irreversible damage but this process takes time to develop. The most important cause of progressive renal damage is from associated infection. Patients with infected urolithiasis will typically present with renal colic type symptoms (colicky pain that is sharp, severe and intermittent with associated nausea and vomiting). Additionally, they will have fever and may demonstrate signs of sepsis (tachycardia, hypotension or shock). Patients with infected stones, particularly those that are obstructing, require hospital admission, intravenous antibiotics and urology consultation. In these patients, sepsis and kidney injury are possible risks and patients may require emergent drainage. Non-contrast CT scan of the abdomen and pelvis is the gold standard for diagnosis of urolithiasis. Antibiotics and follow up (B) is inadequate due to the risk of kidney damage and sepsis. Intravenous hydration (C) may be beneficial in patients with infection and or sepsis but is not adequate management in infected stones. Pain control with follow up (D) is appropriate for the treatment of urolithiasis without signs of infection.

A 19-year-old woman who is G1P0 at 14 weeks presents with lower abdominal pressure and urinary frequency. Workup is negative except a urinalysis, which demonstrates positive leukocyte esterase, nitrite and 15-20 WBC/hpf. What management is indicated? Amoxicillin Ciprofloxacin Levofloxacin Send urine culture and hold treatment

Correct Answer ( A ) Explanation: The patient presents with an urinary infection in pregnancy, which should be treated with an antibiotic that is safe in pregnancy like amoxicillin. Urinary tract infection (UTI) is relatively common in pregnancy (2-7%) and is associated with a number of complications including pyelonephritis, preterm delivery and low birth weight. Asymptomatic bacteriuria is not treated in non-pregnant patients. However, it should always be treated in pregnant women because of the risks of preterm labor, perinatal mortality and maternal pyelonephritis. Appropriate antibiotics include nitrofurantoin, amoxicillin and cephalexin. Trimethoprim-sulfamethoxazole (TMP-SMX) may be used in the first and second trimesters. Patients with pyelonephritis, intractable vomiting or toxic appearance should be admitted for antibiotics. Flouroquinolones (B & C) are useful in the treatment of UTI but are not safe to the fetus in pregnancy as they can cause tendonopathies. The risk of pyelonephritis, preterm labor and perinatal mortality make it inappropriate to delay therapy for urine culture results (D).

A 13-year-old male with sickle cell disease presents with a penile erection for five hours. He complains of severe pain. Which of the following treatments is indicated? Corporeal aspiration Intracorporeal terbutaline Intravenous phenylephrine Phlebotomy

Correct Answer ( A ) Explanation: The patient presents with priapism, a urologic emergency that should be relieved emergently with corporeal aspiration. Priapism describes engorgement of the corpora cavernosa due to either low-flow (more common) or high-flow states. Low-flow priapism is due to decreased venous outflow and is commonly seen in sickle cell disease and leukemia. A number of medications can also cause this disorder. High-flow priapism is generally painless and results from excessive inflow of arterial blood or spinal trauma. Management should be expeditious as prolonged priapism can result in fibrosis and erectile dysfunction. Cavernosal aspiration and irrigation with phenylephrine is the most effective treatment modality. A dorsal nerve block should first be performed followed by aspiration from the corpora cavernosa until the penis detumesces. This can be followed by phenylephrine irrigation into the corpora. Patients with persistent priapism or underlying sickle cell disease or leukemia should be considered for admission.

A 19-year-old man presents to the ED with pain along his penile shaft for the past 7 days. He reports a low-grade fever and myalgias and was sexually active with a new partner 10 days ago. He denies any penile discharge or dysuria. There is no inguinal adenopathy palpated on exam, but he has tender penile lesions, revealed in the image seen above. Which of the following is most likely to be an effective treatment? Acyclovir PO Ceftriaxone IM Doxycycline PO Penicillin G IM

Correct Answer ( A ) Explanation: This is a patient with a primary herpes infection, characterized by a low-grade fever; myalgias; and multiple painful, shallow, tender, genital lesions. These typically follow a 2-7-day incubation period. Patients often do not have adenopathy until the 2nd or 3rd week of illness. Lesions last for 2-4 weeks. Treatment is acyclovir to reduce the duration of symptoms and viral shedding. Ceftriaxone (B) is one of several treatment regimens for chancroid. Although chancroid also presents with painful genital ulcers, 50% of patients also develop a large unilateral, fluctuant lymph node (bubo) 1 week after the ulcers appear. It is often difficult to differentiate herpes from chancroid on exam. However, herpes is orders of magnitude more common in the United States; most cases of chancroid occur in developing countries. There are usually fewer than 100 cases of chancroid reported to the CDC annually. Therefore, in patients such as this one, herpes should be of primary consideration, and acyclovir is more likely to be an effective treatment. Doxycycline (C) is used to treat lymphogranuloma venereum. However, lesions are painless and often go unnoticed. Patients often present in the secondary stage following the disappearance of genital lesions, when they develop painful lymphadenitis. Penicillin (D) would be the appropriate treatment for syphilis. However, the incubation period is typically longer and primary lesions (chancres) are painless with raised edges.

A 59-year-old man presents with 2 days of dysuria, frequency, lower abdominal pain, and chills. He has been eating and drinking normally and is able to urinate, though he has general malaise. He denies penile discharge, testicular pain, sexual activity, a history of sexually transmitted infections, prior urinary symptoms, or weight loss. The patient's abdomen is soft and nontender; his genitourinary exam is normal except that on a gentle rectal exam, he has a boggy, tender prostate. A urinalysis, urine culture, and Gram stain are sent and pending. Which of the following is the most likely cause of this patient's symptoms? Escherichia coli Neisseria gonorrhoeae Peptostreptococcus magnus Pseudomonas aeruginosa

Correct Answer ( A ) Explanation: This is a patient with acute bacterial prostatitis, as evidenced by the development of acute urinary symptoms, malaise, chills, and a boggy, tender prostate. Most of the causative organisms are the same as those that cause urinary tract infections, and E. coli is responsible for more than 80% of cases. Because this patient is over the age of 35 and has no history of sexually transmitted infections, recent contacts, or penile discharge, he is less likely to have N. gonorrhoeae (B), which is much more common in younger men. Gram-positive bacteria, of which P. magnus (C) is an example, are extremely rare causes of prostatitis with the exception of Enterococcus spp. Although P. aeruginosa (D) is often found in prostatitis, it is much less common than E. coli.

Which of the following is a risk factor for developing nephrolithiasis? Glomerulonephrosis Peptic ulcer disease Peripheral vascular disease Previous cholelithiasis

Correct Answer ( B ) Explanation: Most stones are composed of calcium oxalate, alone or in combination with calcium phosphate. Hyperexcretion of calcium is a major contributor of stone formation. Many conditions are associated with hypercalciuria. Patients with peptic ulcer disease tend to ingest large amounts of calcium with food, in addition to absorbed alkali sources and antacids. Glomerulonephrosis (A), peripheral vascular disease (C), and previous cholelithiasis (D) are not risk factors for developing nephrolithiasis. Other risk factors include Crohn's disease, milk-alkali syndrome, hyperparathyroidism, hyperuricosuria, sarcoidosis, recurrent UTI, renal tubular acidosis, gout, laxative abuse, family history, dehydration, hypervitaminosis D, and previous kidney stone.

An 8-month-old boy has a painless swelling on the right side of his scrotum since birth that gets worse with crying. Transillumination of this side of the scrotum reveals increased fluid and the child does not cry on palpation of the testicle which lies in its normal position. Based on these findings, which of the following is the most likely diagnosis? Epididymitis Hydrocele Testicular torsion Varicocele

Correct Answer ( B ) Explanation: A hydrocele is a collection of fluid that accumulates in the tunica vaginalis. Communicating hydroceles result when the upper processus vaginalis fails to be obliterated, leaving an open tract between the peritoneum and the scrotum. The tract is closed in non-communicating hydroceles. Most hydroceles are right sided. Hydroceles may be present at birth, but they usually are painless and worsen with crying or exertion. Hydroceles often resolve spontaneously by the age of 18 months. Examination reveals transillumination and enlargement of the scrotum. Patients with a hydrocele that has persisted for more than 1 year or who are older than 18 months should undergo ultrasonography to ensure that the hydrocele is not a reactive hydrocele caused by testicular tumor or inflammatory process. Testicular torsion (C) presents with acute scrotal pain and swelling, an elevated testicle, and absence of the cremasteric reflex. Testicular torsion has been reported in all age groups, from the developing fetus to the elderly, but is most common in adolescence. Epididymitis (A) is inflammation of the epididymis, which is located along the posterior aspect of the testicle. The most common cause is infectious, and the etiology varies by age. Patients present with a painful, edematous scrotum and tenderness at the epididymis. A varicocele (D) is a collection of venous varicosities of the spermatic veins in the scrotum caused by incomplete drainage of the pampiniform plexus. Left-sided varicoceles account for 85 to 95% of cases but varicoceles are rare in children younger than 10 years. Varicoceles do not transilluminate.

A 32-year-old woman presents with dysuria and lower abdominal discomfort. She is sexually active with one male partner and uses condoms inconsistently. She denies vaginal bleeding or discharge, and her pelvic examination is unremarkable. Urinalysis shows 10 WBC/hpf with positive nitrites. Which of the following is the most likely causative organism? Acinetobacter E. coli Enterobacter Pseudomonas

Correct Answer ( B ) Explanation: Acute cystitis is an isolated bladder infection. Women are more likely than men to have lower urinary tract infections due to shorter urethral length. Urinary tract infections (UTIs) typically result from ascending bacteria from the urethra. Symptomatic patients with a urine culture that grows 102 or 103 colony forming units (CFU)/mL is considered to be infected. The concentration of bacteria in the bladder can increase exponentially in women after sexual intercourse. E. coli is the most common causative organism and is responsible for > 80% of UTIs. Escherichia coli (E. coli) converts nitrates to nitrite, which is a very useful test to confirm the diagnosis of a UTI. The urine nitrate reaction has > 90% specificity. Other organisms causing UTIs include Pseudomonas, Acinetobacter, Enterobacter, Klebsiella, and Proteus species. Acinetobacter (A), Enterobacter (C) and Pseudomonas (D) species do not convert nitrates to nitrites in the urine. It is important to note that while a positive urine nitrite test supports the diagnosis of a UTI, a negative test does not rule it out.

A urine microscopy of a patient with an elevated creatinine reveals muddy brown cellular casts. What is the most likely diagnosis? Acute interstitial nephritis Acute tubular necrosis Nephrolithiasis Pyelonephritis

Correct Answer ( B ) Explanation: Acute tubular necrosis (ATN) is responsible for more than 50% of cases of acute kidney injury in the hospital setting. ATN occurs when there is decreased blood flow to the kidney usually secondary to a drop in blood pressure. The lack of perfusion causes ischemia to the renal tubules, and therefore necrosis. The most significant finding on urine microscopy is muddy brown cellular casts, but additionally, there may be fine granular casts as well as renal tubular epithelial cells. Other urinary labs will reveal a BUN to creatinine ratio of less than 20 as well as a fractional excretion of sodium (FeNa) greater than 2%, both indicative of renal injury. ATN can also be caused by medications, most commonly NSAIDs, ACE inhibitors and angiotensin-receptor blocks (ARBs), which also act to decrease renal blood flow. Acute interstitial nephritis (AIN) (A) is the second most common diagnosis of AKI following ATN. AIN is an acute inflammation of the interstitial space of the kidney most commonly caused by drugs such as antibiotics, antivirals, NSAIDs, PPI's, and diuretics. Urinalysis will reveal hematuria and mild proteinuria. Treatment entails finding and discontinuing the insulting agent, and in some cases, short periods of steroid treatment. Nephrolithiasis (C), or kidney stones, can cause kidney injury by obstructing urine outflow. Patients with this condition are likely to have flank pain, dysuria, and hematuria. Pyelonephritis (D) is an infection of the kidney, which presents with white blood cell casts. This condition can be distinguished from a urinary tract infection by the presence of white cell casts, which are not usually present in an isolated UTI.

A 29-year-old man presents with gradually worsening right testicular pain over the last four days. He is sexually active. On examination, his cremasteric reflex is normal. The testicle has a normal lie. There is tenderness focally over the epididymis. Which of the following tests should be performed next? Epididymal aspiration Gonorrhea and chlamydia swab Rectal exam Urine wet mount

Correct Answer ( B ) Explanation: Epididymitis is the most common inflammatory condition within the scrotum. The epididymis is a tubular structure on the posterior aspect of the testicle where sperm mature. The epididymis becomes infected when retrograde transit of an infection travels from the vas deferens. In men who are sexually active, the organisms of sexually transmitted infections are likely etiologies. For these patients, testing for Gonorrhea and Chlamydia is indicated. For those not sexually active urinary pathogens become more common and a urine culture is indicated. Patients develop gradually increasing pain in the testicle and classically present later for evaluation than patients with torsion who experience severe onset of pain immediately. Men with epididymitis may complain of lower abdominal or flank pain due to inflammation of the vas deferens. Up to 75% of patients have fever. Pain begins in the epididymis and then spreads to the entire testicle. Over time, the ipsilateral hemiscrotum becomes edematous, erythematous and tender. Improvement of pain with elevation of the testicle (Prehn's sign) is often describes as characteristic for epididymitis although it is neither sensitive nor specific. If the clinical history and examination are not clear, an ultrasound of the scrotum will show hypervascularity of the affected epididymis (and testicle if it is involved). Antibiotic therapy is aimed at the most likely organism and recommended for 10-14 days. Complications of epididymitis include orchitis and testicular abscess.

Which of the following is most characteristic of phimosis? Collection of dilated and tortuous veins surrounding the spermatic cord Inability to retract foreskin over glans Inability to return retracted foreskin over glans Painless cyst filled with sperm

Correct Answer ( B ) Explanation: Phimosis is the inability to retract foreskin over the glans penis. It is a complication seen in uncircumcised men. About 50% of boys typically are able to retract their foreskin by 1 year of age and 80% by age 3. Topical estrogen therapy has been reported as successful, but no randomized trials support its use. However, low-potency topical corticosteroid therapy combined with daily prepuce retraction appears effective for phimosis. If retraction of foreskin is unsuccessful, the patient will require circumcision. A varicocele (A) is a collection of dilated and tortuous veins surrounding the spermatic cord. It is most often asymptomatic. Paraphimosis (C) is the inability to return retracted foreskin over the glans. Because vascular engorgement leads to necrosis of the glans, acute paraphimosis needs urgent medical attention. A dorsal slit procedure may be required if other reduction techniques are unsuccessful. Ice packs and plastic wrap may reduce the edema enough to allow manual reduction. A painless cyst filled with sperm (D) is called a spermatocele and can be palpated distinct from the testis. They transilluminate, are generally of no consequence, and do not affect fertility.

A six-year-old girl presents with dark urine and pedal edema and is found to have an elevated blood pressure. She was treated for strep pharyngitis two weeks ago. Which of the following is the next best step? Obtain a blood count Obtain a urinalysis Obtain antistreptolysin titers Obtain serum complement levels

Correct Answer ( B ) Explanation: The patient in this vignette likely has poststreptococcal glomerulonephritis (PSGN). In any patient with concern for renal disease, a urinalysis should be obtained. In patients with renal disease, urine dipstick results show large amounts of blood and protein. Microscopic analysis usually yields leukocytes and white blood cell casts. Red blood cells may appear dysmorphic and can be present with red blood cell casts. Red blood cell casts may or may not be related to PSGN but is pathognomonic for glomerular disease in general. A complete blood count (A) is not usually helpful in the initial workup for renal disease. In this case, the likely diagnosis is PSGN; however, a urine analysis is necessary in the initial workup for any renal disease. Antistreptolysin titers (C) will aid in the diagnosis once renal disease is established. Complement studies (D) are very useful in the diagnosis of PSGN as the consumption of complement is a component in the pathogenesis of the disease. However, urine studies should be obtained prior to any further workup when renal disease is suspected.

A 17-year-old man presents to the ED from a correctional facility complaining of general malaise with nausea and vomiting one day after a weightlifting competition. Vital signs are T 37.2°C, BP 100/65 mm Hg, HR 125, and RR 22. Physical exam reveals an uncomfortable, fatigued male who has diffuse muscle soreness. Urinalysis shows 3+ blood without red blood cells. What is the most important next test to direct the acute management of this patient? Creatine kinase assay Electrocardiogram Electrolyte panel with blood urea nitrogen and creatinine Microscopic urinalysis

Correct Answer ( B ) Explanation: The patient presents with exercised-induced rhabdomyolysis. Although the causes of rhabdomyolysis are manifold, potential complications are independent of etiology. Of all the complications, hyperkalemia is the most concerning and, undiagnosed, can lead to sudden cardiac death. ECG changes indicative of hyperkalemia are thus critical to identify early in the course of management. Hyperkalemia results from impaired calcium transport with increased intracellular calcium accumulation, cellular necrosis, and expulsion of intracellular contents (including potassium) into the bloodstream. Rhabdomyolysis can also lead to acute tubular necrosis and kidney failure, which will exacerbate developing hyperkalemia by decreasing renal potassium clearance. Hyperkalemia is an immediate life-threatening condition that develops shortly after muscle injury. Absent point-of-care electrolyte analyzers, the most rapid way to screen for hyperkalemia is through an electrocardiogram. Rhabdomyolysis is not defined by a specific creatine kinase (A) level. But, in general, a serum CK >5 times the upper limit of normal (a threshold that may differ by lab) is considered indicative of rhabdomyolysis. Although kidney injury can occur at any level, higher CKs correlate with an increased likelihood for the development of acute renal failure. In the absence of cerebral or myocardial infarction, a CK >5 times is diagnostic for serious muscle injury. Urinalysis (D) typically shows brownish discoloration with "large" blood on dipstick but few, if any, red blood cells on microscopic evaluation. This occurs because most dipstick tests cannot distinguish myoglobinuria from hematuria or hemoglobinuria. Protein, brown casts, and renal tubular epithelial cells may also be present. Measures of renal function and an electrolyte panel (C) should be obtained in all patients with suspected rhabdomyolysis. But, as mentioned, waiting for results may lead to a delay in the identification of life-threatening complications. In addition to hyperkalemia, hyperphosphatemia and hypocalcemia may also be seen. Additional (though less worrisome) laboratory abnormalities include elevated uric acid and low albumin.

A patient on peritoneal dialysis presents with fever and abdominal pain. The patient is non-toxic and the abdominal examination is remarkable only for the presence of a peritoneal dialysis catheter. Fluid sent from the peritoneal dialysis catheter contains 153 WBCs. Which of the following managements should be pursued? CT scan of the abdomen and pelvis Intraperitoneal antibiotics Intravenous antibiotics Remove peritoneal dialysis catheter

Correct Answer ( B ) Explanation: The patient presents with peritonitis and should be treated with intraperitoneal antibiotics. The majority of peritoneal dialysis (PD) emergencies are related to the PD catheter with peritonitis being the most common. Peritonitis is caused by bacterial contamination of the dialysate or dialysis tubing during exchange or by infection at the PD catheter exit site extending into the peritoneum. The majority of cases are caused by Staphylococcus aureus and Staphylococcus epidermidis. Direct contamination from the gastrointestinal tract results in a polymicrobial infection. Diagnosis of peritonitis in a patient with a PD catheter is based on the white blood cell count >100 with more than 50% neutrophils. PD-associated peritonitis should be treated with intraperitoneal antibiotics. CT scan of the abdomen and pelvis (A) is not required as there is an explanation of the patient's symptoms. Intravenous antibiotics (C) are unnecessary, more costly and have more side effects. Treatment of PD-associated peritonitis can be done without removal of the catheter (D). Refractory infections may require removal.

A 58-year-old homeless man with a history of end-stage renal disease is brought to the ED via EMS with shortness of breath, fatigue, and malaise for several days, which worsened over the past 2 hours. He denies other symptoms such as a fever, cough, or chest pain. He has T 37.27°C, BP 112/70, HR 114, RR 26, pulse oxygenation 94% on 10 L nonrebreather mask. He is alert and speaking full sentences, though he has diffuse rales on exam. Intravenous access is established, laboratory specimens are sent, and his ECG reveals sinus tachycardia with normal intervals and no ST-T changes. A portable chest X-ray is taken as seen above. Which of the following is most important in the management of this condition? Ceftriaxone Dialysis Morphine Nitroprusside

Correct Answer ( B ) Explanation: This is a patient with chronic kidney disease (CKD) presenting in pulmonary edema from volume overload. Although there are several adjunctive therapies that may be used to temporize this patient's condition—such as bilevel positive airway pressure, nitroglycerin, and furosemide (in patients who retain a significant level of renal function and continue to make urine)—dialysis is the most rapidly effective means of reducing intravascular volume in patients with CKD. It is one of the classic indications for emergent dialysis and should be anticipated early that dialysis would be required for volume-overloaded patients with CKD, even if they receive aggressive medical management. In some cases, differentiating pulmonary edema from pneumonia is challenging, and ceftriaxone (A) is commonly administered. However, this patient's presentation is more consistent with pulmonary edema, and dialysis is therefore more important in his management. Morphine (C) is sometimes given to increase venous capacitance but is not used as first-line therapy. Nitroprusside (D) produces arteriolar dilatation and is used for hypertensive patients in pulmonary edema.

A 23-year-old man presents with burning and itching of the penis. On physical examination, the patient is uncircumcised. There is erythema and inflammation of the distal foreskin and superficial layer of the glans. With retraction of the foreskin there is scant white discharge. What is the appropriate treatment? Ceftriaxone IM Clotrimazole cream Doxycycline Mupirocin ointment

Correct Answer ( B ) Explanation: This patient has balanoposthitis, inflammation of the glans penis (balanitis) as well as the distal foreskin (posthitis). There are multiple causes of balanoposthitis. In younger patients, local irritation from bubble baths is a common cause as well as from soaps and detergents. These cases are treated with topical steroids like hydrocortisone. The etiology may also be infectious with candida as the most common organism. With Candidal infections, a whitish discharge with some eroded plaques may be present. This patient's presentation is suggestive of a Candida infection which is treated with topical antifungal agents such as clotrimazole. Other infectious organisms include anaerobic organisms (treated with topical metronidazole) as well as streptococcal infections. It is uncommon for sexually transmitted infections to cause balanoposthitis. By far the most common contributing factor is poor hygiene. Ceftriaxone administered intramuscularly (A) is the treatment of gonococcal infections. The dose for all gonococcal infections is 250 mg IM. Since sexually transmitted infections are an uncommon cause of balanoposthitis, this medication is not indicated. Doxycycline (C) is used in the treatment of chlamydial infections. A course of doxycycline is prescribed for pelvic inflammatory disease or epididymitis. An uncomplicated urethritis or cervicitis secondary to chlamydia is treatment with a single dose of 1gm of azithromycin. Mupirocin ointment (D) is a topical antibiotic that is used primarily in the treatment of community-associated methicillin resistant Staph aureus infections, a common cause of purulent skin and soft tissue infections in patients presenting to the emergency department.

An 18-year-old sexually active man presents with painful swelling in his groin. He developed painful lesions on his penis one week ago and now has right sided groin swelling. His examination demonstrates multiple penile ulcerations and a large, painful, fluctuant lymph node in the right groin. Which organism is responsible for this infection? Chlamydia trachomatis Haemophilus ducreyi Herpes simplex virus Klebsiella granulomatis

Correct Answer ( B ) Explanation: This patient has chancroid, caused by Haemophilus ducreyi. Patients develop multiple painful genital ulcers and the characteristic inguinal bubo. The ulcerations are sharply demarcated with purulent bases. Fifty percent of patients have lymph node involvement with a unilateral, large, painful and fluctuant node (bubo). H. ducreyi is difficult to culture and the diagnosis is often made clinically. Drainage of the bubo is not routinely recommended as it responds to antimicrobial therapy. Treatment options include: azithromycin 1 g PO, ceftriaxone 250 mg IM, ciprofloxacin 500 mg PO twice daily for three days, or erythromycin 500 mg PO three times daily for seven days. Chlamydia trachomatis (A) causes lymphogranuloma venereum (LGV) in addition to the more common urethritis and cervicitis. Patients develop a transient painless genital ulcer followed by painful inguinal adenopathy. The lymph nodes enlarge above and below Poupart's ligament causing the LGV "groove sign." Treatment is with doxycycline or erythromycin. Herpes simplex virus (C) is the most common cause of painful genital ulcers. It may be associated with bilateral, painful adenopathy. Treatment is with antiviral medications like acyclovir, valacyclovir or famciclovir. Klebsiella granulomatis (D) causes granuloma inguinale characterized by painless genital ulcers. The lesions are highly vascular resulting in a beefy red appearance and bleeding on contact. There is no associated regional adenopathy. Treatment is with doxycycline

A 18-year-old sexually active man presents with testicular pain for 3 days. The pain is constant and is accompanied by fever. Examination reveals a mildly tender right testicle with a normal lie and no urethral discharge. Urinalysis shows 2+ leukocyte esterase, negative nitrites and 10-25 WBCs. What treatment is indicated? Ceftriaxone 250 mg intramuscular Ceftriaxone 250 mg intramuscular and doxycycline 100 mg BID for 10 days Ciprofloxacin 250 mg BID for 3 days Ciprofloxacin 500 mg BID for 14 days

Correct Answer ( B ) Explanation: This patient has signs and symptoms consistent with epididymitis. Epididymitis results from infection of the vas deferens causing edema from passive congestion and inflammation. It usually presents with a gradual onset of scrotal pain unlike torsion, which presents with more abrupt onset of symptoms. 10-30% of patients present with urinary symptoms and 75% report fever. Early in the course, patients present with pain and tenderness localized to the epididymis but symptoms quickly spreads to the adjacent testicle. Classically, Prehn's sign (decrease in pain with elevation of the scrotum) has been taught to be indicative of epididymitis but this sign has a low sensitivity. Urinalysis typically demonstrates signs of infection. Pyuria is common (50-93%). If testicular torsion is suspected, scrotal ultrasound should be pursued. Antibiotic therapy should be tailored based on patient characteristics. In sexually active men, C. trachomatis and N. gonorrhoaea infections are the most common cause of epididymitis particularly in younger men (<35 years of age). Treatment therefore includes ceftriaxone and doxycycline. Older patients are more likely to have urinary tract abnormalities predisposing them to epididymitis from typical urinary tract pathogens (i.e. E. coli) however, if they are sexually active as well, coverage of C. trachomatis and N. gonorrhoaea is also indicated. In addition to bacterial agents, amiodarone can cause epididymitis as it concentrates in the testicle resulting in lymphocytic infiltration. These patients usually do not have fever or pyuria.

A 21-year-old woman presents with urinary frequency and dysuria for two days. Vital signs are within normal limits. Examination reveals mild suprapubic tenderness with no costovertebral angle tenderness. What treatment is indicated? Ceftriaxone Nitrofurantoin Non-contrast CT scan of the abdomen and pelvis Phenazopyridine

Correct Answer ( B ) Explanation: This patient presents with an uncomplicated acute cystitis and can be treated with nitrofurantoin. Urine tract infection (UTI) is an inflammatory response to microorganisms in the urinary tract. Patients will present with dysuria, frequency, urgency, hematuria and suprapubic pain when infection is in the bladder. Cystitis represents infection of the lower urinary tract (pyelonephritis is infection of the upper urinary tract) and is common in women of all age groups but relatively uncommon among young men. Uncomplicated cystitis is defined as inflammation of the bladder in patients in the absence of pregnancy, medical comorbidities or toxic appearance. The most common causative organisms are Escherichia coli and Staphylococcus saprophyticus. First line antibiotic agents for uncomplicated cystitis include nitrofurantoin and trimethoprim-sulfamethoxazole (TMP-SMX). Ciprofloxacin is a common first line drug in regions where resistance to TMP-SMX is greater than 10-20%. Ceftriaxone (A) is a third generation cephalosporin with broad coverage of gram negative pathogens and is widely used for the treatment of pyelonephritis and complicated urinary tract infections. A non-contrast CT scan of the abdomen and pelvis (C) is useful in investigating the presence of renal stones but this patient's history is incompatible with this disease. Phenazopyridine (D) also known as pyridium is used to alleviate the pain, irritation, discomfort, or urgency caused by a cystitis but it does not treat the infection. Its most common side effect is urine discoloration.

A 63-year-old man presents with fever, chills, low back and perineal pain. His pain is increased with urination and he has both frequency and incomplete voiding. On exam, you note a tender prostate. Which of the following treatments is most likely indicated? Ceftriaxone and azithromycin X 1 dose Ciprofloxacin for 4-6 weeks Ciprofloxacin for 7 days Nitrofurantoin

Correct Answer ( B ) Explanation: This patient presents with prostatitis requiring 4-6 weeks of antibiotic therapy with an agent that covers the most common bacterial pathogens. Bacterial prostatitis is primarily caused mainly by Escherichia coli (80%). Other gram negative organisms involved include Klebsiella, Enterobacter, Pseudomonas, and Proteus. Patients present with dysuria, fever, malaise, chills and perineal and low back pain. Rectal examination often reveals a boggy and very tender prostate. Pain is increased with urination and prostatic inflammation often leads to incomplete voiding and frequency. Treatment requires a prolonged course of antibiotics. Appropriate regimens include ciprofloxacin, ofloxacin or trimethoprim/sulfamethoxazole for 4-6 weeks. Fluoroquinolones and third-generation cephalosporins are appropriate for inpatient therapy as well. Ceftriaxone and azithromycin (A) are indicated for the treatment of sexually transmitted infections and cover gonorrhea and Chlamydia. Ciprofloxacin (C) is an appropriate antibiotic for prostatitis but needs to be continued for 30 days. Nitrofurantoin (D) is a first line agent in the treatment of uncomplicated cystitis and is ineffective for bacterial prostatitis.

Which of the following represents a reason for emergent dialysis in a patient with end stage renal disease? Hypophosphatemia Missed dialysis with normal serum potassium level Pulmonary edema responsive to nitroglycerin Severe uremia without symptoms

Correct Answer ( C ) Explanation: Although the decision to institute dialysis in a patient with chronic kidney disease (CKD) is complicated, the indications for emergent dialysis in the Emergency Department are not. Pulmonary edema represents the most common problem patients with CKD present with to the Emergency Department. Patients will typically have worsening dyspnea on exertion or shortness of breath but body weight significantly greater than "dry weight" may be the best indicator. Initial emergency treatment of pulmonary edema in the dialysis patient mimics the treatment in the non-dialysis patient. Supplemental oxygen should be given and nitroglycerin (sublingual or intravenous) started. Regardless of response to these interventions, arrangement for dialysis should be made. Dialysis is the most effective approach to reducing circulating volume when the patient lacks kidney function. Medical treatment may temporarily stabilize a patient while awaiting dialysis. Other emergent indications for dialysis include severe uncontrollable hypertension, hyperkalemia, severe electrolyte or acid-base disturbance, specific overdoses and severe, symptomatic uremia. Hyperphosphatemia, not hypophosphatemia (A), is a trigger for dialysis. Patients may miss a scheduled dialysis treatment (B) without developing complications requiring emergent dialysis. Severe uremia without symptoms (D) does not require emergent dialysis. However, the presence of symptoms including altered mental status or bleeding would qualify the patient for emergent dialysis.

A 57-year-old man presents to the ED for shortness of breath. He has a history of hypertension, diabetes, and end-stage renal disease. His ECG is seen above. Which of the following is true regarding his management? A normal ECG rules out hyperkalemia Calcium chloride administration will decrease the serum potassium concentration Membrane stabilization is a critical first step Repeated doses of IV sodium bicarbonate are recommended until the QRS complex narrows

Correct Answer ( C ) Explanation: Calcium is recommended for the treatment of moderate to severe hyperkalemia (>6.5 mEq/L) where ECG changes are present or the risk of dysrhythmia is present. Calcium's effect is rapid but transient. It provides membrane stabilization within 1 to 3 minutes of administration. Calcium antagonizes the effects of hyperkalemia at the cellular level. In the setting of hyperkalemia, the resting membrane potential is shifted to a less negative value, that is, from −90 mV to −80 mV, which in turn moves the resting membrane potential closer to the normal threshold potential of −75 mV, resulting in increased myocyte excitability. When calcium is given, the threshold potential shifts to a less negative value (that is, from −75 mV to −65 mV), so that the initial difference between the resting and threshold potentials of 15 mV can be restored. The onset of action is less than 3 minutes, and it lasts for about 60 minutes. Calcium gluconate is less tissue toxic than calcium chloride if extravasation occurs. Calcium chloride contains more calcium than the calcium gluconate mixture (6.8 vs. 2.2 mmol in 10 mL) and has a greater bioavailability if the potassium level is significantly elevated. Calcium chloride should be administered through a central line. A normal ECG (A) does not rule out hyperkalemia. Case studies have revealed normal ECGs in patients with potassium levels >8 mEq/L. Calcium (B) does not lower the serum potassium level, but it stabilizes the cardiac membrane by restoring the normal gradient of the resting membrane potential of the cardiac cells. Sodium bicarbonate (D) promotes a shift of potassium into cells. However, sodium bicarbonate should be used with caution when hypertonicity, volume overload, or alkalosis poses a risk to the patient. Administration of an ampule of sodium bicarbonate in this patient could lead to worsening fluid overload and pulmonary edema.

A 48-year-old man presents with fever and an acutely painful scrotum. He has significant pain during testicular palpation. A cremasteric reflex is present and Doppler ultrasonography shows an enlarged, thickened epididymis with increased blood flow to the left testicle. There is no discharge from the urinary meatus. Which of the following is the most appropriate antibiotic for this condition? Ceftriaxone plus doxycycline Doxycycline alone Levofloxacin Penicillin

Correct Answer ( C ) Explanation: Epididymitis occurs most commonly in men between the ages of 14 and 35 years. However, it can occur in any age group. It occurs from an ascending infection from the urethra, prostate, or bladder, and occasionally by hematogenous spread. Epididymitis is characterized by gradually increasing dull, unilateral scrotal pain, fever, and dysuria. Examination usually reveals localized epididymal edema and tenderness (posterior aspect of scrotum), possible testicular tenderness, and a normal cremasteric reflex. Pain may be relieved with testicular elevation (positive Prehn sign). Scrotal pain should be initially evaluated with a color Doppler ultrasound test, and in the case of epididymitis, the typical findings are an enlarged, thickened epididymis with increased blood flow. The most common organisms responsible for epididymitis in those 14 to 35 years-of-age are Neisseria gonorrhoeae and Chlamydia trachomatis. In older individuals (traditionally >35 years of age) and nonsexually active individuals, the Gram-negative rod bacteria (Escherichia, Klebsiella, Enterobacter and Citrobacter species) are most common. Trimethoprim-sulfamethoxazole or a fluoroquinolone such as levofloxacin or ciprofloxacin is the recommended treatment in this age group. Ceftriaxone plus doxycycline (A) is the treatment of choice for suspected orchitis or epididymitis in men between the ages of 14 and 35 years. Doxycycline alone (B) is not recommend as the sole antibiotic for orchitis in any age group. Penicillin (D) is more appropriate for streptococcal or staphylococcal infections, both of which are not common etiologies of orchitis or epididymitis.

A 78-year-old woman is brought to the ED by her daughter with concern for altered mentation and somnolence over the past three days. The patient has a history of hypertension and elevated cholesterol, both well controlled with medication. Her vitals are significant for a blood pressure of 148/72 mm Hg, heart rate of 82 beats per minute, oxygen saturation of 99% on room air, and a temperature of 38.1°C. Physical exam reveals a well-appearing elderly female who is sleeping but easily arousable. Laboratory values show a WBC of 8.2 with 2% bands, hemoglobin of 13.6, platelets of 120,000, sodium of 136, potassium of 3.4, a BUN of 12, and a creatinine of 0.8. Urinalysis is positive for leukocyte esterase, nitrites, and 15 WBC/HPF. Which of the following statements best describes the diagnosis and next step of management? The patient has cystitis; start oral levofloxacin and discharge The patient has cystitis; start oral trimethoprim-sulfamethoxazole and discharge The patient has pyelonephritis; start parenteral ceftriaxone and admit The patient has pyelonephritis; start parenteral trimethoprim-sulfamethoxazole and admit

Correct Answer ( C ) Explanation: In general, symptoms associated with lower UTIs are limited to the genitourinary system and include dysuria, urgency, frequency, hematuria, and suprapubic abdominal pain. In addition to lower urinary symptoms, patients with upper UTIs (pyelonephritis) also develop constitutional symptoms such as fever, vomiting, and malaise and may also have back and flank pain. Therefore, clinical symptoms are often used to differentiate the location of infection. Unfortunately, the correlation between clinical symptoms and the presence and location of the UTI is not exact, and symptoms alone cannot be reliably used to predict the location of infection. Young children and elderly patients, in particular, may have atypical presentations of GU infections. There are multiple treatment options for uncomplicated lower UTI, including single-dose and short-course (three to five days) therapy; local resistance patterns should be consulted when determining a specific antibiotic course. Complicated lower UTI should be treated for seven to ten days. Upper tract UTI should be treated with ten to fourteen days of antimicrobial therapy. Nontoxic patients may be treated as an outpatient. Toxic or pregnant patients, and those with urologic abnormalities, immunocompromised, or inability to tolerate oral intake should be placed on parenteral antibiotics. Ceftriaxone has excellent coverage of E. coli and is generally considered the first line for parenteral treatment of uncomplicated pyelonephritis, but local resistance patterns should be considered when selecting specific agents. Fluoroquinolones are another option.

A 12-year-old boy presents to the ED with swelling and pain in his penis. He reports that over the past 6 hours he has been unable to reduce his foreskin over the glans and that the area has become progressively more swollen. He denies dysuria, discharge, testicular pain, or any other symptoms prior to his foreskin being retracted earlier today. On exam, his foreskin cannot be reduced and the glans is red, swollen, and tender, as seen above. Which of the following is the most likely diagnosis? Balanoposthitis Orchitis Paraphimosis Phimosis

Correct Answer ( C ) Explanation: Paraphimosis is the inability to reduce the foreskin back to its anatomic position over the glans of the penis. It has several causes, including infection, trauma, hair tourniquets, and sexual activity. Distal venous congestion develops and can lead to arterial compromise, necrosis, and gangrene. It is a urologic emergency. You have to call the "paramedics" because "paraphimosis" is an emergency. This patient's absence of penile discharge, testicular symptoms, and sudden development of symptoms after not being able to reduce his foreskin strongly point to paraphimosis as the diagnosis. In addition to manual reduction, substances with a high solute concentration can be used to osmotically draw out fluid from the edematous glans and foreskin prior to manual reduction. Granulated sugar spread over the glans and foreskin for 2 hours has been shown to facilitate manual reduction. Balanoposthitis (A) is inflammation of the glans and foreskin, often caused by infection, trauma, and poor hygiene. Though it is more common in uncircumcised male patients, the foreskin should be reducible, unlike in this case. Orchitis (B) is a testicular infection and does not involve the penis or foreskin. This patient has no testicular symptoms. Phimosis (D) is constriction of the foreskin, which prevents its retraction back from over the glans.

A 20-year-old man presents to the ED after noticing an enlarged mass on the left side of his scrotum. He denies any pain and reports being asymptomatic with no dysuria, frequency, urgency, penile discharge, or weight loss. He has not undergone any trauma to the area and is not sexually active. On exam, a firm, hard, nontender mass is palpated on the left side of the scrotum abutting a testis. On the right side of the scrotum, a testis is palpated without a mass. The testes are nontender. The mass does not transilluminate or change in size when the patient lies flat. Which of the following is the most likely diagnosis? Hydrocele Orchitis Testicular tumor Varicocele

Correct Answer ( C ) Explanation: Testicular tumors are the most common malignancy in young men. The vast majority (95%) of them are germ cell tumors (50% seminomas, 50% mixed types, including teratomas, choriocarcinomas, yolk sac tumors), and a small number (5%) are sex cord stromal tumors. They most commonly present as a painless unilateral scrotal mass palpated discretely from the testis. Hydroceles (A) are fluid collections within the tunica vaginalis and, in some cases, occur due to the presence of a testicular tumor. However, they typically transilluminate and are not firm and hard. Orchitis (B) is a testicular infection common in prepubertal boys. The most typical presentation includes fever and scrotal pain, unlike in this patient. Varicoceles (D) are dilations of the venous varicosities of the spermatic veins. Some 85%-95% of cases occur in the left side, as in this patient, and they may present as "bags of worms" rather than as firm, hard masses.

Which of the following patients requires admission to the hospital? A 19-year-old woman with a 3 mm kidney stone, no hydronephrosis and moderate blood on urinalysis A 19-year-old woman with a 7 mm kidney stone, no hydronephrosis and trace blood on urinalysis A 22-year-old woman with a 5 mm kidney stone, moderate hydronephrosis and >10 white blood cells per high-powered field on urinalysis A 26-year-old woman with a 3 mm kidney stone, no hydronephrosis and >10 white blood cells per high-powered field on urinalysis

Correct Answer ( C ) Explanation: The 22-year-old woman with a 5 mm kidney stone, moderate hydronephrosis and >10 white blood cells per high-powered field on urinalysis has an obstructing and infected kidney stone, which is an indication for hospital admission. Obstruction and infection are the two most common complications of renal calculi. They can both independently lead to significant renal damage, however together can cause significant morbidity and mortality. Complete obstruction alone can lead to irreversible renal damage within 1-2 weeks, including renal calyx rupture. In the urinary tract, renal calculi act as foreign bodies and can easily become infected. Once an infection has developed, the patient is at increased risk for development of pyelonephritis, renal abscess, bacteremia, and gram-negative sepsis. In this clinical scenario, the patient should be resuscitated as indicated, started on broad spectrum intravenous antibiotics and immediate consultation with a urologist should be obtained to determine the need for operative management. The 19-year-old woman with a 3 mm kidney stone, no hydronephrosis and moderate blood on urinalysis (A) has a high likelihood that her stone will pass spontaneously given that most stones under 7 mm pass spontaneously. Although the 19-year-old woman with a 7 mm kidney stone, no hydronephrosis and trace blood on urinalysis (B) has a high likelihood of the stone not passing spontaneously, she has no evidence of concurrent infection or obstruction and can be managed outpatient as long as her pain is controlled and she is tolerating orals. The 26-year-old woman with a 3 mm kidney stone, no hydronephrosis and >10 white blood cells per high-powered field on urinalysis (D) has a mild infection but no evidence of obstruction and therefore can be discharged home on antibiotics and outpatient follow up.

Which of the following is most important in predicting the spontaneous passage of a renal calculus? Age of the patient Calculus location Calculus size Composition of the calculus

Correct Answer ( C ) Explanation: The 3 primary predictors of stone passage without the need for surgical intervention are stone size, stone location, and the degree of patient pain at discharge. The most important factor, however, is calculus size. Approximately 90% of calculi smaller than 5 mm pass spontaneously within 4 weeks. For calculi between 5 and 8 mm, the likelihood of spontaneous passage is 15%. Up to 95% of stones >8 mm become impacted in the genitourinary tract, and lithotripsy or surgical removal is necessary. Renal calculi most commonly occur in young adults and middle-aged men, with nearly 70% of all ureteral calculi occurring between the ages of 20 to 50 years. However, the age of the patient (A) does not affect the rate of spontaneous passage of calculi. Spontaneous passage is more frequent with calculi located below the midureter (B) compared to above the midureter. Patients with well-controlled pain on discharge are less likely to require surgical intervention than those who do not have well-controlled pain. Calcium stones are most common and make up 80% of all stones—of these, 66% are calcium oxalate and 33% are calcium phosphate. The other 20% is made of struvite (magnesium ammonium phosphate), uric acid, and cystine. The composition of the stone (D) has no affect on the rate of spontaneous passage.

Which of the following is one of the most helpful signs to rule out testicular torsion? Presence of a bell-clapper deformity Presence of Prehn's sign (relief of scrotal pain upon elevation of scrotum) Presence of the cremasteric reflex Vertical lie of testicle

Correct Answer ( C ) Explanation: The cremasteric reflex is a superficial reflex elicited by lightly stroking the superior and medial (inner) thigh in a male. The normal response is an immediate contraction of the cremaster muscle that pulls up the testis (>0.5 cm) on the side stroked. The presence of a cremasteric reflex appears to be one of the most helpful signs in ruling out testicular torsion. The cremasteric reflex may be absent in conditions such as testicular torsion, upper and lower motor neuron disorders, spinal injury of L1-L2, and iatrogenic transection of the ilioinguinal nerve during surgery for hernia repair. Despite being one of the most helpful signs, it is important to note that the presence of a cremasteric reflex cannot rule out testicular torsion with 100% certainty. The bell-clapper deformity (A) is an entity in which the tunica vaginalis completely encircles the epididymis, distal spermatic cord, and the testis rather than attaching to the posterolateral aspect of the testis. This causes the testicle to hang free within the tunica vaginalis, in a horizontal fashion. This anatomic abnormality predisposes to spermatic cord torsion. Relief of pain with elevation of a painful testicle represents a positive Prehn's sign (B) and points toward a diagnosis of epididymitis rather than testicular torsion. It is thought that by elevating the painful testicle, the pain of epididymitis improves because the elevation takes the weight of the testis off the epididymal suspension but does not affect the testicle in cases of testicular torsion. However, Prehn's sign is unreliable and should not be used to rule out testicular torsion. Although a vertical lie (D) of the testicle is normal, it does not rule out testicular torsion

A 12-year-old boy presents with penile pain. On exam you note the image above. What management is indicated? Direct application of ice to the skin Follow up with urology Manual reduction Topical clotrimazole

Correct Answer ( C ) Explanation: The patient presents with paraphimosis, a urologic emergency, requiring emergent reduction of the foreskin. Paraphimosis occurs when the foreskin cannot be returned to its anatomic position covering the glans penis. It can be caused by infection, trauma or hair tourniquets. Subsequent venous congestion can lead to arterial compression, penile necrosis and gangrene. The typical presentation is of erythema and engorgement of the penis distal to the obstruction and proximal flaccidity. Treatment is directed at reducing swelling and edema of the foreskin and reducing it to the neutral position. Parenteral anesthesia or a local nerve block may be necessary for manual reduction. Circumferential compression should be held starting at the glans to reduce edema. This should be followed by manual reduction by placing steady pressure on the glans with both thumbs while holding the shaft straight. If these attempts fail, urology should be consulted for dorsal slit procedure. Patients may be discharged home after reduction as long as they are voiding without difficulty and have no signs of severe infection. Although cold water (in a glove or bag) can be useful in reducing edema, ice (A) should not be directly applied. Urology follow up (B) is necessary after reduction has occurred but patients should not be sent home with paraphimosis because of the risk of serious complications. Topical clotrimazole (D) is used to treat fungal infections of the glans like balanitis.

A 5-year-old boy presents to the ED with pain from his penis. Which of the following statements is true regarding this condition? Gentle retraction of the foreskin typically treats the problem It is associated with bulging of the foreskin with urination Manual reduction with firm pressure for 5-10 minutes is the initial treatment Urinary retention rarely occurs

Correct Answer ( C ) Explanation: This patient exhibits paraphimosis. This condition occurs when the proximal foreskin cannot be reduced distally over the glans penis, resulting in distal vascular congestion. This is a true urologic emergency. Physical exam reveals a flaccid proximal penis with erythema and engorgement distal to the obstruction. Initial treatment involves sliding the foreskin distally and manual reduction with firm pressure to the glans for 5-10 minutes to reduce edema. If this is unsuccessful and there is no urologist immediately available, a dorsal slit procedure should be performed by injecting local anesthesia and making a vertical incision of the constricting band. Phimosis is treated by gentle retraction (A) of the foreskin. Patients with phimosis (constriction of the foreskin resulting in the inability to retract the prepuce over the glans) often will note bulging of the foreskin (B) during urination. One complication of paraphimosis is urinary retention (D) from urethral obstruction.

An 18-year-old woman presents with third-degree burns to 20% of her body surface area. During resuscitation, a urinalysis is sent showing granular muddy brown casts. Her BUN and creatinine are 43 mg/dl and 3.3 mg/dl respectively. Total urine output over the first 2 hours is 25 cc. What treatment is indicated to reverse this renal condition? Dopamine Furosemide Intravenous fluids Mannitol

Correct Answer ( C ) Explanation: This patient has signs of acute tubular necrosis (ATN) and should have her hypovolemia addressed with intravenous fluids. ATN is a reversible deterioration of kidney function resulting from a renal insult. The most common cause of ATN is renal hypoperfusion resulting in ischemic injury. This typically occurs after trauma, sepsis or surgery. Additionally, there are a number of iatrogenic nephrotoxins including antibiotics and radiocontrast agents and exogenous toxins including ethylene glycol. Postischemic ATN often occurs from volume loss (GI, skin or kidneys), major burns or severe hemorrhage. Hyperglycemic hyperosmolar nonketotic coma is a common cause of ATN as well. Diagnostic tests will show an increased creatinine (increases as much as 2 mg/dl/day) and a BUN/creatinine ration <10:1. Urinalysis may be positive for heme in 50% of patients and typically show granular muddy brown casts. Treatment aims at reversing decreases in GFR and urine output, minimizing further renal insults and stopping any nephrotoxic agents. Repletion of intravascular volume is central in the management of patients with ATN. Dopamine (A) has not been shown to be effective in increasing renal blood flow or reversing ATN. Furosemide (B) is a loop diuretic which is often effective in increasing urine flow after intravascular volume deficits have been corrected but should not be given before this. Mannitol (D) has a similar effect to furosemide and should not be given until adequate volume has been restored.

A 19-year-old man presents to the clinic with the painless ulcer see in the image above. He reports unprotected intercourse with several partners. What is the recommended first line treatment? Ceftriaxone 250 mg IM x 1 Doxycycline 100 mg PO BID x 7days Levofloxacin 500 mg PO x 7 days Penicillin 2.4 million units IM x1

Correct Answer ( D ) Explanation: Syphilis is caused by the spirochete Treponema pallidum. There are four phases of syphilis: primary, secondary, latent, and tertiary. Primary syphilis is characterized by a painless chancre that begins as a papule and then ulcerates. It is usually solitary and occurs at the site of inoculation. It resolves spontaneously after 2-6 weeks. Patients may also have bilateral, painless, nonfluctuant, inguinal adenopathy. The treatment for primary, secondary, and early (<1year) latent syphilis is benzathine penicillin 2.4 million units IM. Alternative treatments include doxycycline, tetracycline, or erythromycin for penicillin allergic patients. Levofloxacin (C) is not a recommended treatment for syphilis. Ceftriaxone 250 mg IM x1 (A) is the drug of choice in the treatment of gonorrhea (Neisseria gomorrrhoeae). Gonorrhea typical presents with purulent urethral discharge but can be asymptomatic. Doxycycline (B) is an alternative agent for the treatment of syphilis and can be used in those patients who are penicillin allergic.

Which of the following is associated with an increased likelihood of testicular torsion? Age greater than 50 years Epididymitis Fixed testis Undescended testis

Correct Answer ( D ) Explanation: Testicular torsion can occur at any age, but it has bimodal peaks: the first year of life and at puberty (ages 12 to 18). Torsion results from maldevelopment of fixation between the enveloping tunica vaginalis and the posterior scrotal wall, as seen with an undescended testicle (cryptorchidism). Characteristically, the involved testis is aligned along a horizontal rather than a vertical axis. Frequently there is a history of an athletic event, strenuous physical activity, or trauma just before the onset of scrotal pain. The pain usually occurs suddenly, is severe, and is usually felt in the lower abdominal quadrant, the inguinal canal, or the testis. On physical exam, there is loss of the cremasteric reflex. This is observed in nearly 100% of patients with torsion. The cremasteric reflex is a superficial reflex observed in men. This reflex is elicited by lightly stroking the superior and medial part of the thigh. The normal response is an immediate contraction of the cremaster muscle that pulls up the testis on the side stroked. In the setting of testicular torsion, this reflex is absent. The testicle is usually tender and firm and the scrotum is swollen and tender. In cases with a high suspicion of torsion, the patient should be taken to the OR immediately by a urologist. If the diagnosis is equivocal, color Doppler and ultrasound is the best diagnostic modality. Torsion is bimodal (A) with most cases occurring during the first year of life or during puberty. Epididymitis (B) may mimic the symptoms of torsion but is more gradual in onset. It does not predispose to torsion. Nearly 50% of cases arise in men between the ages 20 and 29 years. If untreated, it can lead to orchitis, testicular abscess, and sepsis. Although fixed testes (C) can still torse, it is not associated with an increased likelihood of torsion.

An 18-year-old woman presents with dysuria, hematuria, fever and flank pain. Physical examination reveals a well-appearing woman with moderate costovertebral angle tenderness. Urinalysis is positive for leukocyte esterase, nitrites and 25-50 WBCs. What treatment is indicated? Intravenous ceftriaxone Intravenous ciprofloxacin Nitrofurantoin Oral ciprofloxacin

Correct Answer ( D ) Explanation: The patient presents with pyelonephritis, which can be treated as an outpatient with ciprofloxacin. Urinary tract infection (UTI) is a common problem affecting all age groups and is more common in women than in men. Patients with upper UTI or pyelonephritis often present with fever, malaise, nausea, vomiting, back and flank pain in addition to the dysuria, frequency and hematuria seen in lower UTI. Urinalysis in UTI typically shows leukocyte esterase and nitrites. Microscopy will show at least 10 WBCs. Escherichia coli is the most common bacteria causing UTI, particularly in outpatients. Institutionalized or hospitalized patients may have involvement with atypical bacteria like Proteus, Klebsiella and Enterobacter. Intravenous ceftriaxone (A) and ciprofloxacin (B) are appropriate inpatient treatment for complicated pyelonephritis. Nitrofurantoin (C) is appropriate for the treatment of cystitis but is not active in upper UTI.

A 19-year-old man with diabetes presents with penile pain and dysuria. Physical examination of the penis reveals the image above. The patient states he has had similar episodes in the past. What management is indicated? Azithromycin and ceftriaxone Ciprofloxacin Clotrimazole cream Hydrocortisone cream

orrect Answer ( C ) Explanation: The patient presents with balanitis, which results from inflammation of the glans penis or the glans and foreskin (balanoposthitis). The primary cause is infection, most commonly Candida. Treatment consists of topical antifungal. Balanitis can also be caused by bacterial agents such as group A beta-hemolytic streptococci. Additionally, it may be secondary to irritation, trauma or contact dermatitis. Recurrent balanitis is seen in diabetics from C. albicans. Patients typically complain of pain, discharge, and itching. The glans may appear erythematous and tender to palpation. Systemic symptoms may be present but are unusual. Management of fungal balanitis includes topical antifungal agents such as clotrimazole, adequate hygiene and glycemic control. In patients with no history of diabetes who present with fungal balanitis, diabetes should be considered and a finger stick should be obtained.


Conjuntos de estudio relacionados

Bus 121: Employee Benefits Chapter 5

View Set

Microeconomics Unit 6 - Market Failures

View Set

International Environmental Politics Final

View Set

SAP Review -MAD, Mean, Median, Mode & Range

View Set